Difference between revisions of "2002 AMC 12B Problems/Problem 10"

(See also)
Line 11: Line 11:
  
 
==See also==
 
==See also==
 +
{{AMC12 box|year=2002|ab=B|num-b=9|num-a=11}}
 +
 +
[[Category:Introductory Combinatorics Problems]]

Revision as of 09:52, 5 February 2008

Problem

How many different integers can be expressed as the sum of three distinct members of the set $\{1,4,7,10,13,16,19\}$? $\mathrm{(A)}\ 13 \qquad\mathrm{(B)}\ 16 \qquad\mathrm{(C)}\ 24 \qquad\mathrm{(D)}\ 30 \qquad\mathrm{(E)}\ 35$

Solution

We can make all multiples of three between 1+4+7=12 and 13+16+19=48, inclusive. There are $\frac{48}{3}-\frac{12}{3}+1=13\Rightarrow \boxed{\mathrm{(A)}$ (Error compiling LaTeX. Unknown error_msg) integers we can form.

See also

2002 AMC 12B (ProblemsAnswer KeyResources)
Preceded by
Problem 9
Followed by
Problem 11
1 2 3 4 5 6 7 8 9 10 11 12 13 14 15 16 17 18 19 20 21 22 23 24 25
All AMC 12 Problems and Solutions